« first day (33 days earlier)      last day (541 days later) » 
00:00 - 18:0018:00 - 00:00

6:00 PM
0
Q: Learning and teaching mathematics

Angelo MarkI teach mathematics , both pure and applied mathematics , for students who are yet to be undergraduates (Advanced Level) . I find myself very fond of teaching mathematics , but learning. How learning further more mathematics will help to my career , even I have a good practice and knowledge of ...

0
Q: What is the method to solve this kind of problems in modular arithmetic?

SadStudentx is a three-digit natural number, 2x ≡ 3 (mod 5) 3x ≡ 1 (mod 4) what is the smallest value that x can take? What is the method to solve this kind of questions?

0
Q: Must $\vec{n}$ be a Unit Normal Vector (Stokes' Theorem)?

nmagerkoIf $S$ is an oriented, smooth surface that is bounded by a simple, closed, smooth boundary curve $C$ with positive orientation, then for some vector field $\vec{F}$: $$\oint_C \vec{F} \cdot d\vec{r} = \iint_S {\rm curl} \> \vec{F} \cdot d\vec{S}$$ The latter integral can be written equivalentl...

Questions tend to get more attention when they have a tag for a broad area of mathematics relevant to the question. Some of these tags might fit. (from a bot)Normal Human 21 secs ago
0
Q: Must an algorithm terminate?

user292477I am confused. Sometimes i read about terminating and not terminating algorithms. I almost always read these things in the context of turing machines. This means to me: There are algorithms which terminate and others which don't. But for example the german wikipedia page for algorithm says that ...

0
Q: Euclidean Distance in 4th Dimension

Vari3tyI have found this question in one of my Universities old pass papers and I'm trying to solve this: You have a fishing rod of length 2 and need to ship it in a box which sides are not longer than 1. In spaces Rn of at least which dimensional n will you be able to fit the rod into the box without...

Welcome to Math.SE, Vari3ty. Questions tend to get more attention when they have a tag for a broad area of mathematics relevant to the question. Some of these tags might fit. (autocomment)Normal Human 22 secs ago
0
Q: How do I evaluate the line integral F dot product dr when F = (5*x*y^3)i + (3*x^2*y^2)j?

GoldnameOn the curve C consisting of the x-axis from x=0 to x=4, the parabola y=16−x^2 up to the y-axis, and the y-axis down to the origin. I can't seem to get the right answer. Since the vector field is not conservative, I'll just have to integrate each individual part of the curve. However, the force...

This site uses MathJax formatting of formulas. More tips here. (from a bot)Normal Human 21 secs ago
0
Q: We have the recurrence relation $a_0 = 1$, $a_1 = 2$ and $a_n = 4a_{n-1} - a_{n-2}$. Could you find an odd integer factor of $a_{2015}$?

J.GI just finished the first part of PUTNAM competition this year and I would like to have your opinion. Question : We have the recurrence relation $a_0 = 1$, $a_1 = 2$ and $a_n = 4a_{n-1} - a_{n-2}$. Could you find an odd integer factor of $a_{2015}$? I tried to find an explicit formula for thi...

Tag (contest-math) should not be the only tag a question has. Please add a tag for a subject area to which the question belongs. (autocomment)Normal Human 21 secs ago
0
Q: Compute the length of an intersection curve

TheBlueWizardI am asked too calculate the exact length of the intersection curve of the two equations $$ x^2 = 2 y \\ 3z = xy \\ $$ From the point (0,0,0) to the point (6,18,36). I don't know if I am doing the right thing, but I am starting to isolate $x$ from the first equation and insert it in the second...

Consider adding a tag for a broader subject area to which the question belongs. Some of these tags might fit. (autocomment)Normal Human 21 secs ago
0
Q: Proof for an ACT math question?

idislikepickles123A polynomial in x has m nonzero terms. Another polynomial in x has n nonzero terms, where m

Words such as question do not add information to titles. Please edit the title so that it better describes the specifics of your question. Do not hesitate to make it longer or include a formula if needed. More tips here. (autocomment)Normal Human 21 secs ago
0
Q: Growth rate of two functions

hetajrIt's obvious that $x^2>2x+1$ for $x\ge 3$ - we just observe that for $x\ge3$ the LHS grows much faster than the RHS. But how to determine: how faster does the LHS grow? and conclude from it that the inequality indeed holds?

0
Q: Differential forms, projections

user289139I have a problem with one exercise from differential geometry. I don't even know how to start. Anyone could help with this problem? Let $M$, $N$ be manifolds, $M$ connected. Let $\pi:M\times N \to N $ be a projection of second coordinate. Prove that $k-$form $\omega$ on $M\times N$ is of form ${...

0
Q: Which are some of your favorite questions on this site that still have no complete answer?

Ante PaladinI really do not know if this question is appropriate because maybe some of us will think that question about questions is something silly but I see no reason why it should not be asked. I think that, if this question collects enough answers (although maybe it will collect none), that we will, in...

Tag (soft-question) should not be the only tag a question has. Please add a tag for a subject area to which the question belongs. (from a bot)Normal Human 21 secs ago
0
Q: graphing of two realtions

LegolasWe have the following relations: $S_1=\{(x,y) \in Z^2:x+y>1 and x>0 \}$ $S_2=\{(x,y) \in P^2:x+y>1 and x>0 \}$ We have to make the graph for each occasion.My question is, what is the difference between these two graphs i am a little confused on how to graph them

 
6:38 PM
2
Q: An [exhaustive] list of reasons why this tag is superfluous

5gon12ederI just came about the exhaustive tag. It has no description and a total of 14 questions. Among those, the obvious confusion took place. People have used the tag for the following kind of questions: algorithmic questions about exhaustive search questions about memory exhaustion arbitrary other ...

 
0
Q: I cant visualize these symetries

PersonaAHello I am trying to learn about the symmetries of a regular tetrahedron. I understand the identity and all eight 120 degree rotations that keep one vertex fixed, $(123),(132),(243),(234),(134),(143),(124),(142)$ but I cannot at all understand how to visualize the so called 180 degree rotations ...

0
Q: Check the statement: For all square matrices $A,B$ of order $n$ is $det(AB)=-det(BA)$

display_errorCheck the statement: For all square matrices $A,B$ of order $n$ is $\det(AB)=-\det(BA)$ Is it correct that the statement is not true for all $A,B$?

Questions tend to get more attention when they have a tag for a broad area of mathematics relevant to the question. Some of these tags might fit. (autocomment)Normal Human 21 secs ago
0
Q: Enumeration problem where we have two consecutive events happening

AstromanTwo experiments are to be performed. The first can result in any one of m possible outcomes. If the first experiment results in outcome i, then the second experiment can result in any of ni possible outcomes, i = 1, 2, ... , m. What is the number of possible outcomes of the two experiments? How ...

0
Q: Exponential Properties

TimHere are my steps: $e^{{2\pi i}/100} = (e^{\pi i})^{{2/100}} = ((-1)^2)^{1/100} = 1^{1/100} = 1$. I'm not sure if the normal rules of exponents apply like this if the power is complex.

0
Q: Calculating Area integral

XPenguenI need to calculate the Integral $\int_M1$ where $M:=\{(x,y): (x^2+y^2)^2-2*x*y=0\}$ I just got no clue how to approach this problem. I just know that the integral is the area described by M which looks somewhat like an eight. Can you guys give me a hint in the right decision, because I feel lost...

 
7:03 PM
-2
Q: Can self-censoring end up with a question ban?

RegWe are told The exact formula for the bans is not disclosed, but users are only banned if they have a significant number of ... deleted posts. I'm only starting here, and I know I'll (hopefully) get better. But when someone with astronomic levels of reputation swoops in to point out how pa...

0
Q: Cross posts to Math SE

redelectronsAre cross posts to Math SE allowed? As in almost exactly the same text, just posted there. Of course the sites are meant for difference things, but still just wondering. And if yes, should one link the MO post before posting on Math SE

 
7:14 PM
0
Q: What is the derivative of this integral?

Zequan Wu$$\int_a^{g(x)} f(t) \,dt$$ What is the derivative of this integral?

0
Q: Show that the relation $f\sim g$ if $\int|f-g|=0$ is an equivalence in $L^1(\mathbb{R})$.

Desperate FluffyShow that the relation $f\sim g$ if $\int|f-g|=0$ is an equivalence in $L^1(\mathbb{R})$. What constitutes an equivalence in $L^1(\mathbb{R})$? Any help would greatly appreciated.

0
Q: Change input parameter at each time step - matlab - ode45

ChrisKI am not sure how to change an input parameter 'β' at each time step. My code is below - which gives me an error. Can anybody help please! t = [7 14 21 28 35 42 49 56 63 70 77 84]; for i=1:12; (i) = 0.43e-08 + (4.28e-08 - 0.43e-08)*exp(-0.20*t(i)); end; f = @(t,x) [3494-0.054*x(1)-beta*x...

This site uses MathJax formatting of formulas. More tips here. (autocomment)Normal Human 21 secs ago
0
Q: Moving of 5 units on a curve

TheBlueWizardIf I am at a point (0, 0, 3) and a move of 5 units in the positive direction of $t$ on a curve definite by $$ \begin{align*} x &= 3sin(t) \\ y &= 4t \\ z &= 3cos(t)\\ \end{align*} $$ Where am I ? I am not sure how to start this problem. What does the 5 units in the positive direction of t mean...

0
Q: Maurer-Cartan form left invariance

PtFLet $G$ be a lie group and $\mathfrak{g}$ its lie algebra, that is, the $\mathbb R$-space of left invariant vector fields on $G$. Recall the isomorphism $\mathfrak{g}\simeq T_eG$. The Maurer-cartan form on $G$ is the $1$-form $\omega\in \Omega^1(G; \mathfrak{g})$ defined through the composition ...

Tagged proof-verification. Maurer-Cartan form left invariance
0
Q: Multivariable Calculus - Chain rule - Partial derivatives computation

SademSuppose you are given that f is a differentiable function of two variables u and v. Let g(r,s)=f(r^2-s^2, 2rs). Compute ∇g(r,s) in terms of ∂f/∂u, ∂f/∂v and other functions. I expressed f as f(u,v)=f(r^2-s^2, 2rs) and set u=r^2-s^2 and v=2rs. To calculate the gradient I did: ∇g(r,s)=∂g/∂ri + ∂g...

This site uses MathJax formatting of formulas. More tips here. (from a bot)Normal Human 21 secs ago
0
Q: Covering of hawaiian earring

JavierI'm taking a course on Algebraic Topology and I'm struggling to find the solution to this problem: Let $Y$ be the Hawaiian earring in $\mathbb{R}^2$ and $Y'$ the union on infinite $Y$s moved $3z$ units upward (and downward) with $z \in \mathbb{Z}$ and the line $x=0$ so it is connected. Show a...

 
7:31 PM
0
Q: Should the Black Space tag be removed?

penne12This tag, blank-space, doesn't actually describe anything, and is used for questions that are not related to each other. Is there any way to remove/lock it?

 
0
Q: Which are linear orders on N? Prove an answer

Narmina BaghirovaT,where mTn m<2n V where mVn iff m is odd n is even,or or m,n are even m

0
Q: Help to understand in detail the following proof.

user162343I've been reading the book "Algebra a graduate course" by Isaacs, but I met something in Galois Theory that I didn't understand this proof (18.13) THEOREM. Let $F \subset Ε$ with $[E : F] < \infty$. The following are then equivalent: i. $Ε$ is a Galois extension of $F$. ii. $Ε$ is both separ...

Words such as help do not add information to titles. Please edit the title so that it better describes the specifics of your question. Do not hesitate to make it longer or include a formula if needed. More tips here. (from a bot)Normal Human 21 secs ago
0
Q: How to calculate participant*hours

LucasI'm trying to calculate the number of "participant*hours" for participation in a program. I have the following variables/data: 1) Total # of sessions given throughout the program. 2) Average duration (in hours) of each session. 3) Average # of participants in each session. So, for example, if ...

This site uses MathJax formatting of formulas. Consider adding a tag for a broader subject area to which the question belongs. Some of these tags might fit. More tips here. (autocomment)Normal Human 21 secs ago
0
Q: backward stochastic differential equation

user296172Would you exercices on backward stochastic differential equations? and the link with the PDE? Thank you

0
Q: weak convergence implies strong convergence?

Desperate FluffyI know that strong convergence implies weak convergence, but that the converse is not necessarily true. However, apparently in a finite dimensional Hilbert space it does? Proof?

0
Q: Find the bounded orbits

SalamoConsider $$ \frac{dx}{dt}=y,~~~~~\frac{dy}{dt}=x+x^2-y. $$ Its Hamiltonian with $H(x,y)=\frac{y^2}{2}-\frac{x^2}{2}-\frac{x^3}{3}$ and the equilibria are $$ O_1=(0,0),~~~O_2=(-1,0). $$ The linearization matrix of the system at $O_1$ is $A_1=\begin{pmatrix}0 & 1\\1 & -1\end{pmatrix}$ and so $O_...

Short title. Tagged stability-theory. Find the bounded orbits
 
7:50 PM
-1
Q: Many kernel questions = unstable ubuntu?

Star OSIt seems that, a lot of the new questions here on AskUbuntu are about kernels with "attempted to kill init!", which can mean if there's a new kernel, that kernel could be very unstable? I don't know.

0
Q: Why all the downvotes, I think I followed the rules

jjknuthFirstly, congratulations to everyone participating and having created this community that helps us become better programmers. I am sure there is something missing, that I have not understood. I have made this post following all the instructions and I believe it is pretty clear what is being aske...

 
0
Q: What percentage drop is this?

KeenanCan someone help a student who is terribly bad at math answer this question? Between 1998 and 2014, in New Mexico, Birth rates for teens 18-19 years of age fell from a rate of 108.8 per 1000 to 69.3 per 1,000 females. What percentage had the teen birth rate dropped? Between 1998 and 2014, in New ...

 
8:12 PM
0
Q: Adding and multiplying piecewise functions

EldridgeHow do I add and multiply two piecewise functions? f(x)=x+3 when x<2 and (x+13)/3 when x>2 g(x)=x-3 when x<3 and x-5 when x>3

0
Q: why shouldn't the determinant of jacobian be zero when inverse mapping?

A.Aalipouri have a question about the role of Jacobian in the mapping theorem. I know when i want to map the z=x+iy by f(z)=u(x,y) +iv(x,y), i should put z in the function f and find the relation between x and u , y and v. and i know when inverse mapping i should do them by just w=f^-1(z) . but i faced a p...

Welcome to Math.SE, Eldridge. This site uses MathJax formatting of formulas. More tips here. (from a bot)Normal Human 21 secs ago
0
Q: Help with Linear Regression

tom982My sister just submitted an assignment and got a few questions marked incorrect (electronically) but I've just checked over them and don't believe this to be the case. Can someone either point out where I'm going wrong or confirm my suspicions please? Here is the data (sorry for the formatting!)...

Words such as help do not add information to titles. Please edit the title so that it better describes the specifics of your question. Do not hesitate to make it longer or include a formula if needed. More tips here. (autocomment)Normal Human 21 secs ago
0
Q: How big can a set be?

mickI was reading that the ordinal numbers do not form a set because there are too many of them, instead they form a proper class. Is there a maximum cardinality for a set?

Short title. Short question. How big can a set be?
0
Q: Show that a finite set in R^d has volume 0 (Jordan Regions-Rectangle)

user217189Show that a finite set has volume 0 in R^d (Real Analysis section on Jordan Regions, Rectangle in R^d)

0
Q: Interchanging limits and integrals

darrenCan I interchange the limit ad integral of this function fn(x) := (1 + cosx(sinx)^n)/(n + 1)^2 , integration over [a,b] = [0,pi/2] how is this done I don't understand the theory behind this and looking at example questions has not cleared up how to approach these problems. So how do I solve th...

This site uses MathJax formatting of formulas. More tips here. (from a bot)Normal Human 21 secs ago
0
Q: Black-Scholes derivation

Gabor BakosThis a discounting factor method of deriving the famous BS formula. I have done most of the derivation myself but got stuck on the point mentioned below. I would be grateful to get some advice on how to get from that point to the final answer. (The book I am learning this from the called Asset P...

0
Q: Angle of Cross Product

RatonKeep in mind we are working inside $\mathbb{R}^2$ and $\vec{n}$ and $\vec{d}\in\mathbb{R}^2$ Define $\psi^i_{\vec{n}}$ to be the directional angle of $\vec{n}$ Remember $\cos \left( \psi^i_{\vec{n}} \right) =\frac{n_i}{||\vec{n}||}$. Assuming that $\vec{n}$ and $\vec{d}$ are both in the first...

0
Q: Question on Predicate Calculus

Saul GarciaExercise Description We have been given these exercises on predicate logic, and this is what I have for an answer, I would like to have some feedback to know if I am on the right track, or I have made some mistakes, this is what I've got. MY ANSWERS: 1. ∀x∃y (A(x) -> H(x,y) 2. ∀x∃y∃z ((C(x,...

Words such as question do not add information to titles. Please edit the title so that it better describes the specifics of your question. Do not hesitate to make it longer or include a formula if needed. More tips here. (autocomment)Normal Human 20 secs ago
0
Q: Quadrature formula precision

Ne'meh Abu QamarShow that no matter how nodes and weights of quadrature formula . The formula cannot have precision greater thanShow that no matter how nodes and weights of quadrature formula . The formula cannot have precision greater than 2n+1 2n+1

0
Q: To prove $A\Rightarrow B$ is enough to prove that $\urcorner(A\Rightarrow B)$ is false with a counterexample?

Jelly BellyI want to prove $A\Rightarrow B$ is true. If I prove that $\urcorner(A\Rightarrow B)$ is false with a counter example is it enough?

0
Q: Solving a inequation with ln

KagutsuchiI have to show that the following inequation is true: $\frac{ln(x) + ln(y)}{2} \leq ln(\frac{x+y}{2})$ I transformed it into $\frac{ln(x \cdot y)}{2} \leq ln(x+y) - ln(2)$ because I thought that I better can show the inequation here, but I don't know how to proceed. How can I proceed or am I...

0
Q: derivative of a complex equation

ergonIf I have to find the derivative of a function that requires you to initially use the chain rule and then the rest is a product or quotient, how do I proceed? For example, sin(x^2(x+1)). And also what in case it needs to perform the product/quotient rule and after the chain rule? For example, x^...

Welcome to Math.SE, ergon. This site uses MathJax formatting of formulas. More tips here. (from a bot)Normal Human 20 secs ago
0
Q: How to simplify a messy integrand by integrating by parts

sloppyphysicistI have a function $x(t)$ and a messy integral over $t$, containing $x(t)$, $x'(t)$ and $x''(t)$ which I would like to clean up the integrand by removing the $x''(t)$ by integrating by parts. I have tried defining a PDE by $$ \frac{dz}{dt} = \frac{\partial z}{\partial x}x' + \frac{\partial z}{\par...

Tagged pde, differential-equations. Tagged differential-equations but mentions "partial". How to simplify a messy integrand by integrating by parts
0
Q: How to get the maximum error of linear interpolation of sine in $[0,\frac{\pi}{2}]$

jackI want to get a set of equispaced points in $[0,\pi/2]$ and use piecewise linear interpolation generated by those points to fit the sine function. And I want to determine how many points do I need to put in to generate the linear interpolation fit such that max error in that interval can be guara...

Tagged matlab. [How to get the maximum error of linear interpolation of sine in $[0,\frac{\pi}{2}]$](math.stackexchange.com/q/1561504)
0
Q: Question concerning the differentiation of a scaled function

user285282Consider $u_{\lambda}(t,x) = \lambda^{\alpha} u (\lambda^{\beta}t,\lambda^{\gamma} x)$, for some smooth function $u$. I differentiate this wrt $t$ giving me $$ \partial_t u_{\lambda} = \lambda^{\alpha + \beta} \partial_t u.$$ My first question is simply: can I now treat the arguments of the RHS...

0
Q: Few questions on Combonatrics and Cauchy Frobenius

QualityI am trying to work out some problems and I am wondering if anyone can help to check over my work and help me to understand some parts. I am using CFF to solve these combinatoric questions; First one being, How many different pizzas we can make of 12 slices, if 6 have peppers and 6 don't. My ...

0
Q: Bias of two estimators

user143352I hope someone can help me. I have some trouble calculating the bias of two estimators.Unluckily it is really urgent because I hold a presentation next week. The topic is nonparametric local regression. In order to compare kernel estimators I have to calculate the bias of the following two estima...

0
Q: KhanAcademy nested fractions problem

wswldSo I was given a problem at KA today. They offer you a choice among possible simplified versions of this expression: $\frac{1+\frac{x}{y}}{\frac{x}{y}}$ My solution was: $(1+\frac{x}{y})\div\frac{x}{y}=(1+\frac{x}{y})\cdot\frac{y}{x}=\frac{y}{x}+\frac{\not x}{\not y}\cdot\frac{\not y}{\not x}=...

Title contains problem. KhanAcademy nested fractions problem
0
Q: Proof for a periodic function

KagutsuchiI have to solve the following exercise: The function $f : \mathbb{R} \rightarrow \mathbb{R}$ is a periodic function with $P = 2\pi$ so that $f(x) = f(x + 2\pi)$ is true for all $x \in \mathbb{R}$. Show that there is a $\xi \in \mathbb{R}$ with $f(\xi) = f(\xi + \pi)$. Either I don't understand ...

0
Q: Radius of convergence of ∑nAn(z^n) is the same as radius of convergence of ∑Anz^n

Minus22I know that lim |nAn|^(1/n)=lim |An|^(1/n). But to apply root test, do I need to show lim |nAn|^(1/n-1)=lim |An|^(1/n)?

0
Q: Proof for Pappus's Centroid Theorem with basic geometry?

germangirlHow to prove Pappus's Centroid theorem about volume for a triangle rotated around an external axe? The theorem says that the volume V of a solid of revolution generated by rotating a plane figure F about an external axis is equal to the product of the area A of F and the distance d traveled by it...

0
Q: Modes of Convergence, Real Analysis Folland, Ch 2 Problem 39

Morgan WeissIf $f_n\rightarrow f$ almost uniformly, then $f_n\rightarrow f$ a.e. and in measure. Proof: Since $f_n\rightarrow f$ almost uniformly, then for every $\epsilon > 0$ there is a measurable set $F$ with $\mu(F) < \epsilon$ such that the sequence $\{f_n\}$ converges uniformly on $X\setminus E$. I a...

0
Q: Noetherian commutative rings

mattWe know that a commutative ring is Noetherian if and only if every prime ideal is finitely generated.Why this property is not true for maximal ideals?

Short title. Short question. Noetherian commutative rings
0
Q: harmonic ratio of every four points in conformal mapping

A.Aalipouri know the concept of harmonic functions or the conformal mapping and also i know some characteristics of the conformal mapping( i think ). i have already faced a theorem: "the conformal mapping keeps the harmonic ratio of every four points as a constant value." now, i ask you if you know the pro...

Questions tend to get more attention when they have a tag for a broad area of mathematics relevant to the question. Some of these tags might fit. (from a bot)Normal Human 21 secs ago
0
Q: Homogeneous systems Constant Coefficients Initial Value Problem with Eigenvalue of zero

user5644903I have encountered the following problem involving a homogeneous system of constant coefficients initial value problem, but after determining one of the eigenvalues to be zero, I am not sure how to proceed to find the general solution. $$ \frac {dx}{dt}= \begin{bmatrix} 20 & -20 \...

 
9:24 PM
2
Q: stuccy.com mirroring Stack Overflow

Thomas Wellerstuccy.com seems to mirror the complete Stack Overflow site, even with the logo. This violates at least the trademark guidance rules. I found it via google.de on page 2 of the search results (position 18).

 
0
Q: Solving Rational equations

Richard ChangAt the result, if you get 0=-6, does that mean that there is no solution. If you get 5=5 or 6=6, does that mean any value of x (besides those values that are restricted) are solutions?

Short title. Short question. Solving Rational equations
0
Q: $y' = 3|y|^{2/3}$

bkid100Looking at the separable differential equation $y' = 3|y|^{2/3}$, I see that it is not as easy to solve because it is not Lipschitz continuous. Does anyone know how this would be solved. I believe that there would be 4 solutions, not one like if you simply solved it as a separable equation. Thanks!

0
Q: real analysis question on open sets and the boundary

HatCucumberShow that a set $S ⊆ E$ is open if and only if $S ∩ ∂S = ∅$ This is a question from my real analysis class. I started to show the forward direction by letting $S$ be open and using the definition of openness. I know that $S$ is the space inside and that the boundary of $S$ is all the points on t...

Words such as question are uninformative in titles. Please edit the title so that it better describes the specifics of your question. Do not hesitate to make it longer or include a formula if needed. More tips here. (from a bot)Normal Human 21 secs ago
0
Q: What sort of assumptions should I make to change decidability of a statement?

TurboEuler showed $1+2+3+4+\dots=-1/12$. What sort of assumptions could I force of $1+2+3+4+\dots$ to make the statement 'is $1+2+3+4+\dots=-1/12$? undecidable?

0
Q: Trapezoidal vs Simpsons equation

user50098Consider the formula of Simpson $S(h)$ e Trapezoidal $T(h)$ white $h = (b-a)/2n$. Show that $S(h) = \frac{4T(h) - T(2h)}{3}$

Consider adding a tag for a broader subject area to which the question belongs. Some of these tags might fit. (autocomment)Normal Human 21 secs ago
0
Q: Integration Complication forms

RSparkesEvaluate; $$\int_{0}^{1} \frac{1}{x+\sqrt{1-x^2}} \space dx$$ My main concern is finding the indefinite integral as once i have that the rest is fairly straight forward. Please give a detailed answer with reference to why you made each substitution (what indicated that said substitution would wo...

Short title. Question contains please. Integration Complication forms
0
Q: Order statistics and transformations

SusanAssume random variables X$_1$, ... , X$_n$ and Y$_1$, ..., Y$_n$ are U(0,a)-distributed. Show that Z$_n$ = n*$log\frac{max(Y_{(n)},X_{(n)})}{min(Y_{(n)},X_{(n)})}$ has an Exp(1) Distribution. I've started this problem by setting {X$_1$,...,X$_n$,Y$_1$,...Y$_n$} = {Z$_1$,...,Z$_n$} Then the max(Y...

0
Q: Easy way of deriving or memorizing summation formulas

James SmithMy math professor recently told us that she wants us to be familiar with summation notation. She says we have to have it mastered because we are starting integration next week. She gave us a bunch of formulas to memorize. I know I can simply memorize the list, but I am wondering if there is a qui...

Welcome to Math.SE, Susan. Please don't use (self-learning) tag just because you were self-studying. This tag is only for questions about the process of self-studying. (from a bot)Normal Human 21 secs ago
0
Q: How can I find the last two digits of $2^1000$?

modproHow can I find the last two digits of $2^1000$ mathematically?

0
Q: Cross product identity

RatonDoes cross product have an identity i.e. $\vec{i} \times \vec{somevector}$ = $\vec{somevector}\times \vec{i}$ = $\vec{somevector}$ where $\vec{somevector}\in \mathbb{R^3}$ and is the identity we are looking for.

0
Q: Integration of $d(\cos\theta)$

Michael RobertsNow, I've obviously come across the following during my physics degree: $$ \int{\cos\theta\,d\theta} $$ But I'm starting to see this being introduced to this: $$ \int{d(\cos\theta)} $$ Could anyone just give a foolproof dummies guide to this second expression. For me, it has to equal $\cos\th...

0
Q: Limsup of a sequence

MC989${a_n}$ is a sequence with $limsup(a_n)=1$ as $n$ approaches infinity. I have trouble with these, so I want to verify these (a) $a_n \leq1$ eventually; False (b) $a_n \leq2$ eventually; True (c) $a_n \geq1$ eventually; False If any of these are wrong, could you please provide me with a count...

Short title. Question contains please. Limsup of a sequence
 
10:10 PM
0
Q: Possible solutions for matrix similarity

Mr.VNAssume that matrix $A \in M_n$ is similar to a diagonal matrix $D$ with $0$ or $1$ diagonal entries. I read the textbook and it says that there are $n+1$ such diagonal matrices. However, as I expect, if each entries can select values $0$ or $1$, we should have $(2!)^n$ such kind of matrices. Co...

0
Q: Prove by induction that $3^n +7^n −2$ is divisible by $8$ for all positive integers $n$...

jeremysanchez50Prove by induction that $3^n +7^n −2$ is divisible by $8$ for all positive integers $n$. So far I have the base case completed, and believe I am close to completing the proof itself. Base case:$(n=1)$ $3^1 + 7^1 - 2 = 8/8 = 1 $ Inductive Hypothesis: Assume that $3^n +7^n −2$ is divisible by 8...

0
Q: Probability of being dealt a 5-card hand with 2 pairs in poker

MarkHow do I find the probability of being dealt a 5-cad hand with 2 pairs in poker? The pairs can be any cards, and the fifth card must be another rank than the 2 pairs. I have tried (52/52) x (3/51) x (48/50) x (3/49) x (44/48) = 0.003169 This answer does not seem right. Help?

0
Q: Damped Iteration

DoeFor splitting $A = M-P$ a damped iteration with damping factor $\gamma >1$ is $$x^{k+1} = x^{k} +\gamma M^{-1}r^{k}$$ where $$r^{k} = b-Ax^{k}$$ Show that if $M^{-1}A$ has real, postive eigenvalues, then the damped iteration associated with $M$ converges for sufficiently small $\gamma$. Use this...

Short title. Damped Iteration
This site uses MathJax formatting of formulas. More tips here. (from a bot)Normal Human 21 secs ago
0
Q: I need to fine the volume and surface area of a cone using integration? Just the setup.

user296209enter image description here f(x,y)= 5-$Sqrt{x^2 + y^2}$ Need to know the step-by-step setup using integration. Don't need the final solution. Sorry new to the coding.

0
Q: Ways to deal with generating sets of groups

BihcI find that when I come across questions to do with generating sets of groups, I'm never quite sure how to go about the problem. It's difficult to deal with them purely set-theoretically, as you can't make definitive statements about what they contain (or can you?). I'm interested in how other pe...

 
10:39 PM
0
Q: Claim related to separating hyperplane theorem

White House Fence JumperThere is a claim not directly related to separating hyperplane theorem but uses it in some way (sorry but that is all the context I have). As a condition for applying separating hyperplane theorem it says: Claim: "Suppose that $C$ is a convex set, and $0 \notin C$, then the condition for u...

0
Q: stone-weierstrass assumption

user5458359There are two assumptions in Rudin's proof. The first one: We may assume, without loss of generality, that [a,b]=[0,1]. I searched and was given that a linear transformation t = (x-a)/(b-a) is a continuous mapping of [a,b] to [0,1] and thus we make such assumption. Can anyone expain more? The...

0
Q: Continuous function that is uniformly continuous on a dense subset

Andrei KhLet $(X,d_X)$ and $(Y,d_X)$ be two metric spaces, $E \subset X$ a dense set and $f:X \to Y$ be a continuous function that is uniformly continuous on E. Is $f$ then uniformly continuous on $X$? If yes, is the following proof right? If not, where is the error in my proof? Proof: Let $\epsilon > ...

0
Q: Transformation of random variable y=1/x

EvgueniiLet X have PDF: 1/4 0<x<1 3/8 3<x<5 0 otherwise Let Y=1/X, Find PDF for Y.

0
Q: How many surjective functions are there.

AstromanHow many surjective functions that exists from the set A=(1,2,3,4,5,6) (Domain) towards the set B=(w,x,y,z) (Image) I have no idea on how you do this tbh. Any hints would be helpful.

0
Q: Evaluate the double integral by changing to pol

Chemuser123I experience some difficulty with converting to polar coordinates in integrals. So the question I'm struggling with is 'Evaluate the double integral ∬D x^(6)*y dA where D is the top half of the disc with center the origin and radius 4, by changing to polar coordinates' I'm not sure about solvi...

This site uses MathJax formatting of formulas. More tips here. (from a bot)Normal Human 20 secs ago
0
Q: Upper triangular matrixes

LizGive an example of a square matrix A $\in$ $M_n$ with the following properties. (a) entii(A) $\neq$ 0 for each i $\leq$ n but A is not invertible. (b) entii(A) = 0 for each i $\leq$ n but A is invertible. I know that for an upper triangular matrix the diagonal entries must not equal zero in or...

Short title. Question contains please. Upper triangular matrixes
0
Q: Polygonal Mersenne numbers

redelectronsPolygonal numbers are of the form $\cfrac {n^2(s-2)-n(s-4)}{2}$, where $s$ is the number of sides e.g. when $s=5$ we get pentagonal numbers, and $n$ is which one in order it is i.e. the $n^{th}$ $s$-gonal number. Mersenne numbers are of the form $2^p-1$. We usually speak of Mersenne primes,...

0
Q: How can I use Lagrange polynomial to approximate integral f over [0.1]?

dlalz$\int_0^{1}f(x)dx$ is approximated by Af(1/3)+Bf(2/3) I would like to derive above formula using Lagrange interpolation polynomial. How should I start?

Short question. [How can I use Lagrange polynomial to approximate integral f over [0.1]?](math.stackexchange.com/q/1561681)
0
Q: Interchanging differentiable functions

darrenI have a function fn(x)=x^n where fn:(0,1/2)→R I am trying to find wether the differentiable function can interchange the differentiation and the limit and also determine the pointwise limit of {fn} and {f'n} I don't understand how to approach these kinds of problems, I have answered que...

Consider replacing (analysis) with a more specific tag for the relevant branch of analysis. (autocomment)Normal Human 21 secs ago
0
Q: Folland Exercise 3.26, sum of mutually singular measures regular implies each measure regular

DougI am stuck on Exercise 3.26 in Folland's Real Analysis: Let $\mu$ and $\nu$ be positive Borel measures on $\mathbf{R}^n$ such that $\mu + \nu$ is regular. Prove that $\mu$ and $\nu$ are regular. It is immediate that $\mu$ and $\nu$ are finite on compact sets. So it remains to show for, ...

0
Q: Evaluating $\sum_{w_1\in\Omega_1}...\sum_{w_n\in\Omega_n} P(w_1)...P(w_n)$

mavaviljThis is in the context of product of probability functions, but the summation done here should be general. I have trouble seeing the rule happening when $$\sum_{w_1\in\Omega_1}...\sum_{w_n\in\Omega_n} P(w_1)...P(w_n)$$ $\implies$ $$\sum_{w_1\in\Omega_1}P(w_1)\sum_{w_2\in\Omega_2}...\sum_{w_n\...

0
Q: Let $f$ be a relation from $\mathbb{Z}_{4}$ to $\mathbb{Z}_{8}$ given by $f([x])=[x^2+6x]$. Prove or disprove that this is a well-defined function

B.V.Let $f$ be a relation from $\mathbb{Z}_{4}$ to $\mathbb{Z}_{8}$ given by $f([x])=[x^2+6x]$. Prove or disprove that this is a well-defined function. I've attempted to both prove it and find a counterexample, but I've reached a dead-end in either case. For the proof, this is what I have so far...

Tagged proof-verification. [Let $f$ be a relation from $\mathbb{Z}_{4}$ to $\mathbb{Z}_{8}$ given by $f([x])=[x^2+6x]$. Prove or disprove that this is a well-defined function](math.stackexchange.com/q/1561706)
0
Q: Proving that if $f:D \to \mathbb{R}$ is continuous and compact {$f(x_n)$} is a Cauchy sequence.

MC989If $f:D \to \mathbb{R}$ be continuous and let {${x_n}$} be a Cauchy sequence in $D$. Assuming $D$ is closed and bounded, show that {$f(x_n)$} is a Cauchy sequence.

0
Q: $A_k=\{(w_1,w_2,...,w_n)\in \Omega^n\}: w_i=1 \text{ for exactly k indices}$ $\implies$ $|A_k|={n \choose k}$

mavaviljCould someone elaborate how the following implication is seen: $$A_k=\{(w_1,w_2,...,w_n)\in \Omega^n\}: w_i=1 \text{ for exactly k indices}\}$$ $$\implies|A_k|={n \choose k}$$

0
Q: Partial Differential equation

Mike∂u^2/∂t^2=∂u^2/∂x^2 u(0,t)=0 ∂u/∂x(1,t)=1 u(x,0)=0 ∂u/∂t(x,0)=0 I solve the heat equation by using Laplace transformations and found that L^(-1){sinh(sx)/s cosh(s)} Can anyone help me to find u(x,t) using the residues ?

This site uses MathJax formatting of formulas. More tips here. (autocomment)Normal Human 20 secs ago
0
Q: Very ample divisors over non -algebraically complete field

ParakeeFor a projective scheme $V$ over an algebraically-closed field $k$ it is a well-known fact fact that a base-point-free linear system of divisors is very ample iff it separated $k$-points and tangent vectors. I have not seen this proven for non-algebraically closed fields, and suspect that it actu...

0
Q: Prove that a simple graph with $2n$ vertices and $n^2 +1$ edges contains a triangle for $n \ge 2$

Marius KüpperProve that a simple graph $G$ with $2n$ vertices and $n^2 +1$ edges contains a triangle for $n \ge 2$. I see it for $n = 2$ or $n = 3$ ... , but I fail to generalize it.

 
00:00 - 18:0018:00 - 00:00

« first day (33 days earlier)      last day (541 days later) »